Đến nội dung

le_hoang1995 nội dung

Có 342 mục bởi le_hoang1995 (Tìm giới hạn từ 29-04-2020)



Sắp theo                Sắp xếp  

#329064 với: $a^2+b^2+c^2=3$ CM: $\sum \frac{a^2}{b^4+c}...

Đã gửi bởi le_hoang1995 on 25-06-2012 - 17:59 trong Bất đẳng thức và cực trị

mình đưa ra 2 bài cho mọi người cùng chém nhá :D
Bài 1:
với: $a+b+c=3$
CM: $\sum \frac{a}{b+c^2}\geq \frac{3}{2}$

Áp dụng BĐT BCS ta có
$$VT\geq \frac{(a+b+c)^2}{ab+bc+ca+a^2c+c^2b+b^2a}$$
Ta cần chứng minh $ab+bc+ca+a^2c+c^2b+b^2a\leq 6$

Thật vậy, theo AM-GM ta có

$$ab+bc+ca+a^2c+c^2b+b^2a\leq ab+bc+ca+a^{3}+b^{3}+c^{3}$$
$$=ab+bc+ca+(a+b+c)[a^{2}+b^{2}+c^{2}-(ab+bc+ca)]+3abc$$
$$=3(a^{2}+b^{2}+c^{2})-2(ab+bc+ca)+(a+b+c)abc$$
$$\leq 3(a+b+c)^{2}-8(ab+bc+ca)+\dfrac{(ab+bc+ca)^{2}}{3}=27-8(ab+bc+ca)+\dfrac{(ab+bc+ca)^{2}}{3} $$
$$=\dfrac{(ab+bc+ca)^{2}-24(ab+bc+ca)+81}{3} =\dfrac{(ab+bc+ca-12)^{2}}{3}-21$$
$$\leq\dfrac{(\dfrac{(a+b+c)^{2}}{3}-12)^{2}}{3}-21=6$$
Thay vào BĐT trên ta được
$$VT\geq \frac{(a+b+c)^2}{ab+bc+ca+a^2c+c^2b+b^2a}\geq \frac{9}{6}=\frac{3}{2}$$
Dấu "=" xảy ra khi $a=b=c=1$



#329074 với: $a^2+b^2+c^2=3$ CM: $\sum \frac{a^2}{b^4+c}...

Đã gửi bởi le_hoang1995 on 25-06-2012 - 18:42 trong Bất đẳng thức và cực trị

Nếu thế thì đến chỗ $$ab+bc+ca+a^2c+c^2b+b^2a\leq 27-8(ab+bc+ca)+\frac{(ab+bc+ca)^2}{3}=27-8t+\frac{t^2}{3}$$
Với $t \leq 3$
Khảo sát hàm số $f(t)$ sẽ suy ra $f(t) \leq 6$



#332102 Với mọi $x_{1},x_{2},...x_{n}\in R;a_{1},a_{2},...a_{n}>0$...

Đã gửi bởi le_hoang1995 on 05-07-2012 - 12:44 trong Bất đẳng thức - Cực trị

Bài toán: Với mọi $x_{1},x_{2},...x_{n}\in R;a_{1},a_{2},...a_{n}>0$ hãy chứng minh bất đẳng thức sau:

$\frac{1}{n-1}\left ( x_{1}+x_{2}+...+x_{n} \right )^2\leq \left ( \frac{a_{1}^{n+1}}{S-a_{1}} +...+\frac{a_{n}^{m+1}}{S-a_{n}}\right )\left ( \frac{x_{1}^{2}}{a_{1}^{m}}+ \frac{x_{2}^{2}}{a_{2}^{m}}+...+\frac{x_{n}^{2}}{a_{n}^{m}}\right )$
Trong đó: $m,n \in N;m\geq 2$ và $S=a_1+a_2+...+a_n$




P/s: Bác nào sửa giùm e cái tiêu đề

Cái số mũ đầu tiên của VP có vấn đề rồi, giải như sau. Theo Chebusep 2 lần và CS ta có
$$\frac{a_1^{m+1}}{S-a_1}+...+\frac{a_n^{m+1}}{S-a_n}\geq \frac{1}{n}.(a_1^{m+1}+...+a_n^{m+1}).\left ( \frac{1}{S-a_1}+...+\frac{1}{S-a_n} \right )$$
$$\geq \frac{1}{n}.\frac{1}{n}.(a_1+...+a_n).(a_1^m+...+a_n^m).\frac{n^2}{\sum_{k=1}^{n} (S-a_k)}$$
$$=(a_1+...+a_n).(a_1^m+...+a_n^m).\frac{1}{(n-1).S}=\frac{a_1^m+...+a_n^m}{n-1}$$
Như vậy áp dụng CS ta được $$VP\geq \frac{a_1^m+...+a_n^m}{n-1}.\frac{(x_1+x_2+...+x_n)^2}{a_1^m+...+a_n^m}=VT$$
Dấu bằng xảy ra khi $a_1=a_2=...=a_n$ và $x_1=x_2=...=x_n$



#333470 Trận chung kết MSS 2012 - Hiệp 3 - Phương trình

Đã gửi bởi le_hoang1995 on 09-07-2012 - 07:25 trong Thi giải toán Marathon cấp THCS 2012

Bác Thịng thông cảm! Đánh nhầm thiếu dấu mà!
Xưng đệ nhưng chưa chắc đã là đệ, mình xưng vậy cho thân thôi.
Cuối cùng cũng xog MSS, phù

Bạn này hay nhầm quá, Thịnh không phải Thịng ( mà để gõ được chữ này unikey hơi vất :lol: ) Tiếng Việt không có đuôi "ing"



#342793 Trong bất đẳng thức Jensen suy rộng nếu như f(x) lồi trên toàn bộ trục số có...

Đã gửi bởi le_hoang1995 on 02-08-2012 - 12:18 trong Tài liệu, chuyên đề, phương pháp về Bất đẳng thức

Bất đẳng thức Jensen suy rộng đây:
$(\lambda_1+\lambda _2+...+\lambda _n)f(\frac{\lambda _1x_1+\lambda _2x_2...+\lambda _nx_n}{\lambda_1+\lambda _2+...+\lambda _n}) \leq \lambda _1f(x_1)+\lambda _2f(x_2)+...+\lambda _nf(x_n)$

Với phát biểu trên thì $\lambda _i\in (0,1)$ tuy nhiên vẫn phải kèm rằng buộc $\lambda _i>0$ nếu $\lambda _i < 0$ thì cần phải có thêm một điều kiện nào đó(về sự sắp sếp của dãy $\lambda _1,\lambda _2,....,\lambda _n$ hoặc sự sắp xếp của dãy $x_1,x_2,....,x_n$. Em sẽ làm rõ hơn về vấn đề này

Theo em biết thì BĐT Jensen tổng quát là cái này
Cho f là hàm liên tục và lồi trên $\mathbb{I}$. Nếu $x_k \in \mathbb{I}; t_k \in(0;1)$ và $ t_1+t_2+...+t_n=1$ thì ta có
$$t_1f(x_1)+t_2f(x_2)+...+t_nf(x_n)\geq f(t_1x_1+t_2x_2+...+t_nx_n)$$
Chứng minh nó bằng tiếp tuyến
Còn BĐT trên của anh là hệ quả của cái này, nó chỉ cần lớn hơn 0 là được, không cần đk trong khoảng $(0;1)$



#535392 Topic về Bất đẳng thức, cực trị THCS

Đã gửi bởi le_hoang1995 on 29-11-2014 - 21:08 trong Bất đẳng thức và cực trị

Nhờ các bạn giúp mình bài này:
Cho các số dương x, y, z thoả mãn x + y + z = 1. Tìm MIN của P = ax2  + by2 + cz2 (với a, b, c >0).

Áp dụng BĐT Cauchy-Schwarz ta được

$(ax^2+by^2+cz^2)\left ( \frac{1}{a}+\frac{1}{b}+\frac{1}{c} \right )\geq (x+y+z)^2=1$
$\Rightarrow ax^2+by^2+cz^2\geq \frac{abc}{ab+bc+ca}$

Đẳng thức xảy ra khi và chỉ khi $x+y+z=1$ và $ax=by=cz$

Đặt $ax=by=cz=t$, thay vào $x+y+z=1$, ta được

$t.\left ( \frac{1}{a}+\frac{1}{b}+\frac{1}{c} \right )=1$
$\Rightarrow t=\frac{abc}{ab+bc+ca}$
$\Rightarrow \left\{\begin{matrix}
x=\frac{bc}{ab+bc+ca}\\ y=\frac{ca}{ab+bc+ca}
\\z=\frac{ab}{ab+bc+ca}

\end{matrix}\right.$




#304437 Topic trao đổi bài

Đã gửi bởi le_hoang1995 on 15-03-2012 - 19:20 trong Các dạng toán THPT khác

Em làm tiếp bài 26, một cách không hay lắm, nhưng chưa nghĩ ra cách khác được.

Bước 1 chứng minh bằng quy nạp toán học rằng

$1^3+2^3+3^3+...+n^3=\frac{n^2.(n+1)^2}{4}$. Cái này không khó lắm.

Bước 2, chứng minh:

$C_{3}^{3}+C_{4}^{3}+C_{5}^{3}+...+C_{n+1}^{3}=C_{n+2}^{4}$

Xét $f(x)=(x+1)^3+(x+1)^4+...+(x+1)^{n+1}=\frac{(x+1)^3.[(x+1)^{n-1}-1}{(x+1)-1}=\frac{(x+1)^{n+2}-(x+1)^3}{x}$ (cấp số nhân)

Cân bằng hệ số tương ứng của $x^3$ trong 2 vế, suy ra

$C_{3}^{3}+C_{4}^{3}+C_{5}^{3}+...+C_{n+1}^{3}=C_{n+2}^{4}$

Bước 3.

Rút gọn lại biểu thức, ta cần chứng minh:

$\frac{n^2.(n+1)^2}{4}=6C_{n+2}^{4}+C_{n+1}^{2}$

$6C_{n+2}^{4}+C_{n+1}^{2}=6\frac{(n+2).(n+1).n.(n-1)}{4!}+\frac{(n+1).n}{2!}$

$6C_{n+2}^{4}+C_{n+1}^{2}=\frac{(n+2).(n+1).n.(n-1)}{4}+\frac{(n+1).n}{2}$

$=\frac{n.(n+1)}{2}.[\frac{(n+2).(n-1)}{2}+1]=\frac{n^2.(n+1)^2}{4}$

Suy ra ĐPCM.

Các bài còn lại em vẫn chưa có lời giải, mong mọi người chỉ giáo :lol:



#303923 Topic trao đổi bài

Đã gửi bởi le_hoang1995 on 13-03-2012 - 06:40 trong Các dạng toán THPT khác

Sao không ai làm vậy, mình làm trước bài 25.

Ta có $k.k!=(k+1).k!-k!=(k+1)!-k!=f(k+1)-f(k)$ trong đó $f(k)=k!$

Như vậy $S_1=f(2)-f(1)+f(3)-f(2)+f(4)-f(3)+...+f(n+1)-f(n)=f(n+1)-f(1)=(n+1)!-1$

b) Ta có $(k^2+k+1).k!=(k^2+2k+1).k!-k.k!=(k+1)^2.k!-k.k!=(k+1).(k+1)!-k.k! =g(k+1)-g(k)$

Tương tự câu a $S_2=g(2)-g(1)+g(3)-g(2)+...+g(n+1)-g(n)=g(n+1)-g(1)=(n+1).(n+1)!-1$.



#303044 Topic trao đổi bài

Đã gửi bởi le_hoang1995 on 09-03-2012 - 06:35 trong Các dạng toán THPT khác

Em đang học đến Lim, có bài này nghĩ ra cách rồi mà lằng nhằng quá, muốn tìm thêm cách khác, post lên đây nhờ các anh chị giải quyết giúp.

Bài 24: Tính:

a) $\lim_{x\rightarrow 0}\frac{(1+mx)^n-(1+nx)^m}{x^2}$

b) $\lim_{x\rightarrow 1}\frac{m}{1-x^m}-\frac{n}{1-x^n}$

c) $\lim_{x\rightarrow 0}(1+sin3x)^{\frac{1}{x}}$



#303265 Topic trao đổi bài

Đã gửi bởi le_hoang1995 on 10-03-2012 - 02:28 trong Các dạng toán THPT khác

Câu b em cũng làm giống như vậy. Còn đây là câu a.

$\lim_{x\rightarrow 0}\frac{(1+mx)^n-(1+nx)^m}{x^2}$

Theo nhị thức Niu tơn ta có
$(1+mx)^n-(1+nx)^m=\sum C_{n}^{k}(mx)^k-\sum C_{n}^{k}(nx)^k=[1+mnx+\sum_{k=2}^{n}C_{n}^{k}(mx)^k]-[1+mnx+\sum_{k=2}^{m}C_{m}^k(nx)^k]$

$=x^2[\sum_{k=2}^{n}C_{n}^{k}m^kx^{k-2}-\sum_{k=2}^{n}C_{m}^{k}n^kx^{k-2}]$

Như vậy $\lim_{x\rightarrow 0}f(x)=\lim_{x\rightarrow 0}[\sum_{k=2}^{n}C_{n}^{k}m^kx^{k-2}-\sum_{k=2}^{n}C_{m}^{k}m^kx^{k-2}]=C_{n}^{2}m^2-C_{m}^{2}n^2$

Câu c hóa ra chưa học giới hạn e.

Tiếp tục.

Bài 25. Tính

$A=1.1!+2.2!+3.3!+...+n.n!$

$B=(1^2+1+1).1!+(2^2+2+1).2!+...+(n^2+n+1).n!$

Mấy câu này em muốn tính trực tiếp nhưng không ra, dùng Maple để tính rồi quy nạp thì không hay cho lắm :(

Bài 26. chứng minh rằng:

$1^3+2^3+3^3+...+n^3=6(C_{3}^{3}+C_{4}^{3}+C_{5}^{3}+...+C_{n+1}^{3})+C_{n+1}^{2}$

Bài 27.Cho các số nguyên dương m,n. Chứng minh rằng $\frac{(m.n)!}{(m!)^n.n!}$ là một số nguyên.

Bài 28. cho số $A=2000.2001.2002$. Tìm số các ước số của A không chia hết cho 1001.

Bài 29. Cho a là số thực thỏa mãn
$f(x)=\frac{2008}{2009}cosx+\frac{2011}{2008}cos(x-a)+1\geq 0,\forall x\in R$

Chứng minh rằng $f(x)\leq 3,\forall x\in R$



#295503 Topic các bất đẳng thức lớp 8 hay dùng và các bài toán BĐT

Đã gửi bởi le_hoang1995 on 23-01-2012 - 09:40 trong Bất đẳng thức và cực trị

Tổng quát nhỏ cho bài toán này: Cho $a,b,c>0$ và $n$ là số tự nhiên dương. Chứng minh:
$$\frac{{{a^n}}}{{b + c}} + \frac{{{b^n}}}{{c + a}} + \frac{{{c^n}}}{{a + b}} \geqslant \frac{{{a^{n - 1}} + {b^{n - 1}} + {c^{n - 1}}}}{2}$$
Khi $n=1$ thì ta được BĐT Nesbit 3 biến.
-------------------------------------------
P/s: Không biết có tổng quát được cho $m$ biến không :D


Bài này em dùng Chê-bư-sép là ra.

Giả sử $a\geq b\geq c\Rightarrow \frac{1}{b+c}\geq \frac{1}{c+a}\geq \frac{1}{a+b}$

$$\Rightarrow VT\geq \frac{1}{3}(a^{n-1}+b^{n-1} +c^{n-1})(\frac{a}{b+c}+\frac{b}{c+a}+\frac{c}{a+b})\geq \frac{1}{3}(a^{n-1}+b^{n-1} +c^{n-1})\frac{1}{3}(a+b+c)\frac{9}{2(a+b+c)}=VP$$

Tổng quát : BĐT trên vẫn đúng cho m số $a_i$ (ở dưới mẫu có dạng $S-a_i$ trong đó $S=\sum_{i=1}^{20}a_i$

Chứng minh vẫn chỉ dùng Chê-bư-sép, đây là ý kiến chủ quan của em



#341613 Topic bất đẳng thức THCS (2)

Đã gửi bởi le_hoang1995 on 29-07-2012 - 20:32 trong Bất đẳng thức và cực trị

Bài 484:Cho $a,b,c\in R^{+}$ thỏa $a+b+c=1$.Chứng minh:
$$\frac{a}{a+\sqrt{a+bc}}+\frac{b}{b+\sqrt{b+ca}}+\frac{c}{c+\sqrt{c+ab}}\leq 1$$

$$VT=\sum \frac{a}{a+\sqrt{a+bc}}=\sum \frac{a}{a+\sqrt{a(a+b+c)+bc}}$$
$$=\sum \frac{a}{a+\sqrt{(a+b)(a+c)}}\leq \sum \frac{a}{a+\sqrt{ac}+\sqrt{ab}}$$
$$=\sum \frac{\sqrt{a}}{\sqrt{a}+\sqrt{b}+\sqrt{c}}=1$$



#332329 Topic bất đẳng thức THCS (2)

Đã gửi bởi le_hoang1995 on 05-07-2012 - 22:14 trong Bất đẳng thức và cực trị

Thêm một bài duy trì topic.
Bài 415. Cho a,b,c dương, chứng minh
$$\sqrt{\frac{2a}{b+c}}+\sqrt{\frac{2b}{c+a}}+\sqrt{\frac{2c}{a+b}}\geq \frac{(a+b+c)^2}{a^2+b^2+c^2}$$



#294748 Topic bất đẳng thức THCS (2)

Đã gửi bởi le_hoang1995 on 19-01-2012 - 23:04 trong Bất đẳng thức và cực trị

Nhưng quan trọng là $a+b-c,b+c-a,a+c-b$ chưa chắc đã hoàn toán >0 có lẽ bài này nên thêm điều kiện $a,b,c$ là 3 cạnh tam giác, còn nếu ko bài này có một cách khác là dùng AM-GM cho $n$ số cũng ra


Không đâu, BĐT của mình xảy ra khi x+y$\geq$0 mà.

$Ta thấy (a+b-c)+(b+c-a)=2b>0$ thế nên có thể áp dụng được



#335084 Topic bất đẳng thức THCS (2)

Đã gửi bởi le_hoang1995 on 13-07-2012 - 00:19 trong Bất đẳng thức và cực trị

Tiếp tục nhé.
Bài 442 . Cho $a,b,c > 0$ và $a^3+b^3+c^3\leq 3$. Chứng minh rằng:
$$\frac{ab}{\sqrt{3+c}}+\frac{bc}{\sqrt{3+a}}+\frac{ca}{\sqrt{3+b}}\leq \frac{3}{2}$$
Bài 443 . Cho $a,b,c>0$ và $abc=1$. Chứng minh:
$$\sum \frac{1}{2(a^3+1)+b^3+c^3}\leq \frac{1}{2}$$
Bài 444 . Cho $a,b,c>0$ và $a+b+c=1$. Chứng minh rằng:
$$2\left ( \frac{ab}{c+ab}+\frac{bc}{a+bc}+\frac{ca}{b+ca} \right )\geq \sqrt{ \frac{ab}{c+ab}}+\sqrt{\frac{bc}{a+bc}}+\sqrt{\frac{ca}{b+ca}}$$



#297670 Topic bất đẳng thức THCS (2)

Đã gửi bởi le_hoang1995 on 01-02-2012 - 13:03 trong Bất đẳng thức và cực trị

Thêm một cách cho bài 233, sử dụng Holder, không biết có phù hợp với THCS không, mình post cho các bạn tham khảo.

$VT=\sum \frac{x^2}{(2y+z)(2z+y)}=\sum \frac{3x^3}{3x(2y+z)(2z+y)}\geq \sum \frac{3x^3}{(\frac{3(x+y+z)}{3})^3}=\sum \frac{3x^3}{(x+y+z)^3}=\frac{3(x^3+y^3+z^3)}{(x+y+z)^3}$

Cần chứng minh $\frac{3(x^3+y^3+z^3)}{(x+y+z)^3}\geq \frac{1}{3}\Leftrightarrow (1+1+1)(1+1+1)(x^3+y^3+z^3)\geq (x+y+z)^3$

Đúng theo Holder.



#294736 Topic bất đẳng thức THCS (2)

Đã gửi bởi le_hoang1995 on 19-01-2012 - 21:49 trong Bất đẳng thức và cực trị

Điều kiện x nằm trong [-1;2]
Từ giả thiết ta có: $x=y\sqrt{4-x^2}+xy\Leftrightarrow \frac{x}{y}=\sqrt{4-x^2}+x$
Bài quán quy về tìm GTLN, GTNN của $\sqrt{4-x^2}+x$
Mình làm được GTLN
Áp dụng BĐT B.C.S $\sqrt{4-x^2}+x\leq \sqrt{(1+1)(4-x^2+x^2)}=2\sqrt{2}$
Còn tìm min hồi trước có học mà hình như quên rồi :P


Mình nghĩ x nằm trong đoạn -2 đến 2 chứ.

Sử dụng BĐT $\sqrt{a}+\sqrt{b}\geq \sqrt{a+b}$ cho a,b không âm. Dấu bằng xảy ra khi a=0 hoặc b=0 hoặc cả hai.

$S=\sqrt{4-x^{2}}+\sqrt{(x+2)^{2}}-2\geq \sqrt{4-x^{2}+(x+2)^{2}}-2=\sqrt{8+4x}-2\geq \sqrt{8-4*2}-2=-2$

Dấu bằng xảy ra khi x=-2



#332284 Topic bất đẳng thức THCS (2)

Đã gửi bởi le_hoang1995 on 05-07-2012 - 21:23 trong Bất đẳng thức và cực trị

Bài 412: Chứng minh rằng với mọi số thực dương $x,y,z$, ta luôn có:

$\frac{x^{3}+y^{3}+z^{3}}{3xyz}+(\frac{ab+bc+ac}{a^{2}+b^{2}+c^{2}})^{2}\geq 2$.

Trần Quốc Anh


Đặt $A=a^2+b^2+c^2-ab-bc-ca$, dễ thấy $A \geq 0$. Ta có
$\frac{a^3+b^3+c^3}{3abc}-1=\frac{(a+b+c).A}{3abc}$
Và $1-\left ( \frac{ab+bc+ca}{a^2+b^2+c^2} \right )^2=\frac{(a^2+b^2+c^2)^2-(ab+bc+ca)^2}{(a^2+b^2+c^2)^2}=\frac{A.(a^2+b^2+c^2+ab+bc+ca)}{(a^2+b^2+c^2)^2}$
BĐT cần chứng minh tương đương với $$\frac{a^3+b^3+c^3}{3abc}-1\geq 1-\left ( \frac{ab+bc+ca}{a^2+b^2+c^2} \right )^2$$
$$\Leftrightarrow \frac{A.(a+b+c)}{3abc}\geq \frac{A(a^2+b^2+c^2+ab+bc+ca)}{(a^2+b^2+c^2)^2}$$
Ta cần có $$\frac{a+b+c}{3abc}-\frac{a^2+b^2+c^2+ab+c+ca}{(a^2+b^2+c^2)^2}\geq 0$$
Thật vậy $$\frac{a+b+c}{3abc}-\frac{a^2+b^2+c^2+ab+c+ca}{(a^2+b^2+c^2)^2}\geq \frac{a+b+c}{3abc}-\frac{2(a^2+b^2+c^2)}{(a^2+b^2+c^2)^2}$$
$$=\frac{a+b+c}{3abc}-\frac{2}{(a^2+b^2+c^2)}=\frac{(a+b+c)(a^2+b^2+c^2)-6abc}{3abc(a^2+b^2+c^2)}\geq \frac{9abc-6abc}{3abc(a^2+b^2+c^2)}=\frac{1}{a^2+b^2+c^2}>0$$
Vậy BĐT được chứng minh. Dấu bằng xảy ra khi $A=0$ hay $a=b=c$



#327497 Topic bất đẳng thức THCS (2)

Đã gửi bởi le_hoang1995 on 21-06-2012 - 09:21 trong Bất đẳng thức và cực trị

Bài 390 Chứng minh rằng với $a,b,c>0$ ta có
$$\frac{a+b}{c}+\frac{b+c}{a}+\frac{c+a}{b}\geq 3+\frac{(a^2+b^2+c^2)(ab+bc+ca)}{abc(a+b+c)}$$



#294745 Topic bất đẳng thức THCS (2)

Đã gửi bởi le_hoang1995 on 19-01-2012 - 22:44 trong Bất đẳng thức và cực trị

Bài 118:Cho $a,b,c>0$ thỏa mãn $a+b+c=3$. $CMR$
$\sum \frac{5-3bc}{1+a}\geq ab+bc+ac$


Tiếp tục bài 118.

$\frac{5-3bc}{1+a}\geq \frac{5-\frac{(b+c+1)^{3}}{9}}{1+3}=\frac{45-(4-a)^{3}}{9(1+a)}$

Ta chứng minh $\frac{45-(4-a)^{3}}{9(1+a)}\geq a$

$\Leftrightarrow \frac{a^{3}-12a^{2}+48a-19}{9(a+1)}-a\geq 0$

$\Leftrightarrow \frac{a^{3}-21a^{2}+39a-19}{9(a+1)}\geq 0$

$\Leftrightarrow \frac{(a-1)^{2}(a-19)}{9(a+1)}\geq 0$ Đúng.

Tương tự 2 BĐT nữa rồi cộng lại ta có :

$\sum \frac{5-3bc}{1+a}\geq a+b+c=3=\frac{(a+b+c)^{2}}{3}\geq ab+bc+ca$

Dấu bằng xảy ra khi a=b=c=1



#295038 Topic bất đẳng thức THCS (2)

Đã gửi bởi le_hoang1995 on 21-01-2012 - 14:55 trong Bất đẳng thức và cực trị

Bài 130: Tìm $min$ của biểu thức:
$\frac{a^{6}}{b^{3}+c^{3}}+\frac{b^{6}}{a^{3}+c^{3}}+\frac{c^{6}}{b^{3}+a^{3}}$.
Trong đó $a,b,c$ là các số thực dương thỏa mãn điều kiện $a+b+c=1$
zzZZzzZZzzZZzzZZzzZZzzZZzzZZzzZZzzZZ.zZmanZz


Một cách nữa cho bài này, sử dụng chê-bư-sép
$\frac{a^{6}}{b^{3}+c^{3}}+\frac{b^{6}}{a^{3}+c^{3}}+\frac{c^{6}}{b^{3}+a^{3}}\geq \frac{1}{3}(a^{3}+b^{3}+c^{3})(\frac{a^{3}}{b^3+c^3}+\frac{b^{3}}{c^{3}+a^{3}}+\frac{c^{3}}{a^{3}+b^{3}})$

$\geq \frac{1}{3}*\frac{(a+b+c)^{3}}{9}*\frac{3}{2}= \frac{1}{18}$



#339324 Topic bất đẳng thức THCS (2)

Đã gửi bởi le_hoang1995 on 23-07-2012 - 20:03 trong Bất đẳng thức và cực trị

Mình làm bài này nếu đề là $\frac{3}{2\sqrt[3]{(abc)^2}}$ :D
Áp dụng bất đẳng thức $Cauchy-Schwarz$ dạng $Engel$ và $AM-GM$ ta có:
$\frac{a}{bc(b+c)}+\frac{b}{ac(a+c)}+\frac{c}{ab(a+b)}=\frac{a^2}{abc(b+c)}+\frac{b^2}{abc(a+c)}+\frac{c^2}{abc(a+b)}\geq \frac{(a+b+c)^2}{2abc(a+b+c)}=\frac{a+b+c}{2abc}\geq \frac{3}{2\sqrt[3]{(abc)^2}}$

Bổ sung thêm một cách nữa nhé, dạng của nó khiến ta nghĩ đến BĐT Chebusep.

Không mất tổng quát, giả sử $ a \ge b \ge c$, khi đó cách bộ sau đơn điệu cùng chiều

$\left ( \frac{a}{b+c};\frac{b}{c+a};\frac{c}{a+b} \right )$ và $\left ( \frac{1}{bc};\frac{1}{ca};\frac{1}{ab} \right )$

Theo BĐT Chebusep và Nesbit
$$\frac{a}{bc(b+c)}+\frac{b}{ac(a+c)}+\frac{c}{ab(a+b)}\geq \frac{1}{3}.\left ( \frac{a}{b+c}+\frac{b}{c+a}+\frac{c}{a+b} \right ).\left ( \frac{1}{bc}+\frac{1}{ca}+\frac{1}{ab} \right )\geq \frac{1}{3}.\frac{3}{2}.\frac{3}{\sqrt[3]{(abc)^2}}=\frac{3}{2\sqrt[3]{(abc)^2}}$$



#295031 Topic bất đẳng thức THCS (2)

Đã gửi bởi le_hoang1995 on 21-01-2012 - 14:44 trong Bất đẳng thức và cực trị

Quy nạp theo cách này mình thấy vẫn chưa ổn lắm. Không biết ý kiến các bạn như thế nào ?

Theo mình thì B2 của bạn mới chỉ giả sử BĐT đúng với k số, chứng minh BĐT cũng đúng với k+1 số, nhưng bạn đã nhầm lẫn khi dùng chung kí hiệu nên mới suy ra $x_{k+1}=1$.
Nếu mình nhớ không lầm thì đây là bài tập trong SBT toán 11 có thì phải



#294742 Topic bất đẳng thức THCS (2)

Đã gửi bởi le_hoang1995 on 19-01-2012 - 22:14 trong Bất đẳng thức và cực trị

Bài 109 :
Cho $a,b,c>0$: CMR
\[{\left( {a + b - c} \right)^n} + {\left( {b + c - a} \right)^n} + {\left( {c + a - b} \right)^n} \ge {a^n} + {b^n} + {c^n}\]


Ta chứng minh BĐT sau: $\frac{x^{n}+y^{n}}{2}\geq \left ( \frac{x+y}{2} \right )^{n}$ với x+y>=0 có thể quy nạp dễ dàng

Vậy $\frac{(a+b-c)^{n}+(b+c-a)^{n}}{2}\geq \left ( \frac{a+b-c+b+c-a}{2} \right )^{n}=b^{n}$

Tương tự với a và c rồi cộng 3 BĐT lại ta có ĐPCM

Tại sao tui với bạn trên lại cùng chí hướng thế nhỉ, hihi :icon6: , mỗi tội post chậm hơn



#326892 Topic bất đẳng thức THCS (2)

Đã gửi bởi le_hoang1995 on 19-06-2012 - 00:14 trong Bất đẳng thức và cực trị

Bài 386: Cho 3 số $a,b,c$ thỏa $a+b+c=6$. Chứng minh: $$\frac{a}{\sqrt{(b+2)(b^2-b+2)}}+\frac{b}{\sqrt{(c+2)(c^2-c+2)}}+\frac{c}{\sqrt{(a+2)(a^2-a+2)}}\geq \frac{3}{2}$$

Bài này ý tưởng chính dùng cauchy ngược dấu. Theo BĐT AM-GM ta có
$$\sum \frac{a}{\sqrt{(b+2)(b^2-b+2)}}\geq \sum \frac{a}{\frac{b^2+4}{2}}=\sum \frac{2a}{b^2+4}$$
Vậy ta sẽ chứng minh
$$\sum \frac{4a}{b^2+4}\geq 3$$
Thật vậy $$\sum \frac{4a}{b^2+4}=\sum a-\frac{ab^2}{b^2+4}\geq \sum a-\frac{ab^2}{4b}=\sum a-\frac{ab}{4}$$
$$=a+b+c-\frac{ab+bc+ca}{4}\geq 6-\frac{(a+b+c)^2}{4.3}=6-\frac{6^2}{4.3}=3$$
ĐPCM. Dấu bằng xảy ra khi $a=b=c=2$
Bài 387 Cho $a,b,c$ là các số thực dương. Chứng minh rằng:
$$\frac{a}{(b+c)^2}+\frac{b}{(c+a)^2}+\frac{c}{(a+b)^2}\geq \frac{9}{4(a+b+c)}$$
Bài 388 Cho các số thực dương $a,b,c$ thỏa mãn $ab+bc+ca=1$. Chứng minh rằng:
$$\frac{1}{abc}+\frac{4}{(a+b)(b+c)(c+a)}\geq \frac{9\sqrt{3}}{2}$$